subject
Physics, 16.07.2020 21:01 PrincessKeliah5538

A pair of long, rigid metal rods each of length L, lie parallel to each other on a perfectly smooth table. Their ends are connected by identical, very light conduction springs of force constant k each and negligible unstretched length. If the current I runs through the circuit, the springs will stretch. At what separation will the rods remain at rest? Assume that k is large enough so that the separation of rods will be much less than L. Express your answer in terms of I, L, k and appropriate constants (μo,π).

ansver
Answers: 1

Another question on Physics

question
Physics, 22.06.2019 05:10
Which situation will have the highest resistance? a.long wire and high temperatureb.short wire and high temperaturec.long wire and cold temperaturedshort wire and low temperature
Answers: 2
question
Physics, 22.06.2019 12:20
What is the coefficient of kinetic friction μk between the block and the tabletop?
Answers: 1
question
Physics, 22.06.2019 23:30
How is the ideal mechanical advantage of a wheel and axle calculated?
Answers: 1
question
Physics, 23.06.2019 03:00
Are good conductors of heat also good conductors of electricity? explain your answer.
Answers: 2
You know the right answer?
A pair of long, rigid metal rods each of length L, lie parallel to each other on a perfectly smooth...
Questions
Questions on the website: 13722367